Università
Discussioni su temi che riguardano Università della categoria Matematicamente
Algebra, logica, teoria dei numeri e matematica discreta
Discussioni su Algebra astratta, Logica Matematica, Teoria dei Numeri, Matematica Discreta, Teoria dei Codici, Algebra degli insiemi finiti, Crittografia.
Analisi matematica di base
Quando all'Università i problemi con la matematica tolgono il sonno, cerca aiuto qui
Analisi Numerica e Ricerca Operativa
Discussioni su Analisi Numerica e Ricerca Operativa
Analisi superiore
Discussioni su calcolo di variabile complessa, distribuzioni, Trasformata di Fourier, Teoria della misura, Analisi funzionale, Equazioni alle derivate parziali, Calcolo delle Variazioni e oltre.
Fisica, Fisica Matematica, Fisica applicata, Astronomia
Discussioni su argomenti di Fisica, Fisica Matematica, Astronomia e applicazioni della Fisica
Geometria e Algebra Lineare
Discussioni su problemi, esercizi e teoremi che riguardano la geometria, l'algebra lineare e la topologia
Informatica
Discussioni su argomenti di Informatica
Ingegneria
Discussioni su tematiche di ingegneria che non trovano collocazione specifica negli altri forum
Matematica per l'Economia e per le Scienze Naturali
Discussioni su argomenti di matematica per le scienze economiche e finanziarie, la teoria dei giochi, e per le scienze naturali
Pensare un po' di più
Spazio dedicato a problemi che vanno al di là dei semplici temi d'esame o degli esercizi standard.
Statistica e Probabilità
Questioni di statistica, calcolo delle probabilità, calcolo combinatorio
Domande e risposte
Ordina per
In evidenza

salve! ho un dubbio su come calcolare la velocità. Avevo pensato la conservazione dell'energia considerando che Ki=0 e Uf=0 perchè raggiunge la quota che ho scelto come riferimento ma viene energia cinetica negativa quindi non credo vada bene.
Questo procedimento è corretto oppure devo procedere diversamente?

Si faccia riferimento alla immagine sotto. Non capisco bene perche' la traduzione formale evidenziata in giallo sia "proibita".
In altre parole, se considero il predicato P(x)
P(x): "There is a person x having the properties that x is a student in this class and x has visited Mexico.”
e considero il caso particolare in cui
a: Person who is not a student in this class
Perche' P(a) deve essere falsa? E' una convenzione o c'e' qualcosa che mi sfugge?

Si consideri la disuguaglianza di Clausius. E' chiaro che se il ciclo e' reversibile allora vale l'uguaglianza. E' vero che l'uguaglianza vale solo nel caso reversibile?
La stessa questione e' affrontata nel link riportato sotto, ma senza risposte convincenti.
https://physics.stackexchange.com/questions/518206/how-does-fermi-jump-to-this-conclusion-in-clausius-inequality?noredirect=1&lq=1

Ciao. Forse è una domanda stupida, ma... Siano (fcolon A_1 o B) e (gcolon A_2 o B) due monomorfismi, e sia (A_1cong A_2). È vero che (f) e (g) sono equivalenti? nel senso, è vero che esistono (hcolon A_1 o A_2) e (kcolon A_2 o A_1) che fanno commutare gli ovvi diagrammi?

Ciao a tutti,
vi scrivo perché avrei una domanda banale, ma comunque alla quale vorrei dare un chiarimento una volta per tutte.
Ho iniziato da poco gli esercizi su continuità e derivabilità della funzioni in due variabili in Analisi2.
Ecco l'esercizio che ho
$ { ( (x^2y)/(x^2+y^4) ),( 0 ):} $ rispettivamente per la coppia $ (x,y)!= (0,0) $ e $ (x,y)= (0,0) $ .
Ora, ho visto che il prof usa tre ''metodi'' risolutivi: il passaggio alle coordinate polari con $ rho $ e $ theta $ , le ...

Salve a tutti, sono stato assalito da un dubbio atroce e allo stesso tempo banale studiando la serie $ sum_(a = 0)^oo x^a/(a!) $ , ossia l'esponenziale; mi è sembrato impossibile poter calcolare e^0=1 se non si considera 0^0=1, che però ovunque vedo che danno come forma indeterminata. Mi è sfuggito qualcosa o effettivamente c'è una convenzione per cui 0^0=1 e dunque il calcolo con la serie di e^0 ha senso?

Salve a tutti,
stavo facendo esercizi sull'applicazione del teorema di L'Hospital, e per controprova cercavo di risolvere gli stessi limiti con i metodi algebrici tramite limiti notevoli e stime asintotiche. Ecco allora il problema, ad esempio con questo limite:
\[ \lim_{x\to 0}\frac{x\sin x+2\cos x - 2}{x^2\sin^2 x} \]
Ora con L'Hospital viene, secondo il libro, $-1/12$, e quindi qualcosa non va nel mio calcolo:
\begin{align*}
\lim_{x\to 0}&\frac{x\sin x+2\cos x - 2}{x^2\sin^2 ...

Ciao a tutti potete aiutarmi con questo esercizio?
Sia $B_n={(\rhocost, \rhosint) : \rho \in [2,3], t \in [n\alpha, (n+1)\alpha]}$, con $\alpha= pi/(138\sqrt(3))$
Calcolare, se esiste $lim_{n->infty}int_{B_n}(x^3)/(2+cos^2(nxy))dxdy$.
Ho provato a riscrivere la funzione come combinazione lineare di funzioni caratteristiche per integrare su $R^2$, poi passando in coordinate polari ho provato a cercare una funzione sommabile che maggiorasse la successione ma non l'ho trovata. Come potrei fare? Grazie!

Ciao, vorrei cercare di chiarire un dubbio su versori polari.
Ho compreso che le coordinate polari sono dei parametri che mi indicano la posizione del punto con una dupla (cioè una coppia di parametri/coordinate che indicano la distanza dall'origine e l'angolo di apertura rispetto all'asse iniziale).
Il dubbio è invece correlato ai versori polari, cosa sono? Non riesco a concepirli nel senso che sono dei vettori diretti uno come la direzione radialeel'altro tangente al movimento (arco) creato ...

Ciao, vorrei inquadrare meglio un passaggio del libro di fisica. Contestualmente sto seguento analisi 2 e non riesco bene a far combaciare le "due visioni". Vorrei capire in veste analitica questa derivata direzionale fatta solo in "d":
(Inserisco l'immagine per far capire il disegno)
Dovrei avere una derivata direzionale in un sistema cartesiano quindi riesco a vedere la formula del gradiente che: "gradiente per versore per d = derivata direzionale".
Però non capisco la ...

Buongiorno, stavo svolgendo esercizi sui limiti e mi e' venuto un dubbio.
Mi sono ritrovato in una situazione in cui potevo ricondurmi a $cosx/x$ per finire il limite ($x->0$) in fretta, ho quindi pensato a quanto potesse valere un simile limite, sono quindi partito da $\lim_{x\rightarrow 0} (1-cosx)/x^2=1/2$ per sapere che $\lim_{x\rightarrow 0} (1-cosx)/x=0$ e ho quindi pensato che $\lim_{x\rightarrow 0} (1-cosx)/x=\lim_{x\rightarrow 0} 1/x -cosx/x$ potesse ricondurmi a $\lim_{x\rightarrow 0} cosx/x=\lim_{x\rightarrow 0} 1/x$ che sarebbe quindi $+oo$.
E' illegale fare una cosa del genere o ...

Ho un dubbio su un passaggio del professore durante la spiegazione del metodo risolutivo di una EDO a variabili separabili.
Il problema di Cauchy è il seguente
$ { ( y'(t)=py(t)-k ),( y(0)=y_0\in \mathbf{R^+} ):} $
Ora, se $py_0-k!=0$ si ottiene
$\frac{y'(t)}{py(t)-k}=1$
$\Rightarrow \frac{1}{p} \ln|py-k|=t+c$
e a questo punto va discusso il modulo, e il prof dice:
se $py_0-k>0$ allora abbiamo $\ln(py-k)$ altrimenti ecc ecc
Bene, quello che non capisco è perchè si discute il modulo e si richiede $py_0-k!=0$ considerando ...

Un cubetto di ferro subisce una spinta di 145N quando è completamente immerso in acqua. Quanto misura lo spigolo del cubo?
(24,5cm)
avendo la spinta $S=145N$ e la densità dell'acqua $d=1000kg / m^3$ come bisogna procedere? Devo calcolare il volume? non mi è chiarissimo questo argomento

Ciao, mi scuso per la domanda banale, ma dovrei dimostrare che se una funzione non è iniettiva, allora la cardinalità dell'immagine è minore della cardinalità dell'insieme di partenza. È facile verificarlo "graficamente" ma non riesco a formalizzarlo.
Più formalmente, sia
$f: A \to B$ una funzione non iniettiva, allora
$|Im(f)| < |A|$
Grazie mille per l'attenzione e vi ringrazio in anticipo

Buonasera!
Spero di scrivere nella sezione giusta!
Sono alla ricerca di dispense e/o libri che introducano e diano le proprietà relative al fascio/fibrato canonico (canonical bundle) $omega_X$ su una varietà proiettiva e a ciò che in inglese si chiama "line bundle".
Non avendo mai incontrato queste nozioni, ho riportato anche i nomi in inglese, sperando che qualcuno mi possa aiutare.
Grazie mille!

salve a tutti mi sto preparando per dare l'esame di analisi 1 ma c'è questo esercizio che proprio non ho idea di come fare, dati $A = {x ∈ R | (x + 5 −(√(x + 2)))/(e^x +2) >= 0}$ e $B = {x ∈ R | (log|x+2|)/((e^-x)(x-2)^3) <=0} $
determinare A U B ed A intersezione B questa è la traccia ma non ho idea di come fare vi ringrazio per l'aiuto

Ciao a tutti, da poco ho iniziato il corso di ingegneria informatica e tra i vari corsi che sto affrontando vi è il corso di algebra lineare e matematica discreta (a mio avviso il più duro). il mio problema è capire le dimostrazioni e i teoremi sugli spazi vettoriali, basi vettoriali, vettori linearmente dip. e indipendenti etc. Potete darmi qualche consiglio su come approcciare una materia così astratta? Grazie per l'aiuto

Ragazzi i diodi raddrizzatori trasformano una corrente alternata in continua. Ma se io volevo fare la stessa operazione ma senza l uso di diodi fisici, in che altra modo ci potrei riuscire in wireless ??

Sul libro c'è l'esempio di un' equazione differenziale autonoma $ddot x (t)=g(x)$ ; viene riscritta in un sistema del primo ordine di 2 eq. differenziali che corrisponde al campo:
$f([(x),(y)])=[[y],[-g(x)]]$
mi chiedo se questa sia un'equazione di 2 variabili o una sola.
$x$ e $y$ sottintendono $x(t)$ e $y(t)$, quindi mi verrebbe da dire che sia una sola variabile dato che dipende solo da $t$, ossia $f: RR rightarrow RR^2$.
O invece è una ...
Vorrei chiedervi se potreste farmi vedere come dimostrare il seguente risultato:
Sia $a !=0$ e $x_0 in R^+$, allora $lim_(x->0^+)x^a = +infty (a<0), =0 (a>0)$ e $lim_(x->+infty)x^a = +infty (a>0), =0(a<0)$
Purtroppo in tutti i libri di analisi 1 che ho consultato manca la dimostrazione.
Vi ringrazio per l'aiuto!